Renal Physiology Flashcards

1
Q

what is the normal range of Na+?

A

135-145

How well did you know this?
1
Not at all
2
3
4
5
Perfectly
2
Q

what is the normal range of K-?
what is the major concern of high potassium levels?

A

3.5-5
increases risk of arrhythmia

How well did you know this?
1
Not at all
2
3
4
5
Perfectly
3
Q

what is the normal range of Cl-?

A

96-106

How well did you know this?
1
Not at all
2
3
4
5
Perfectly
4
Q

what is the normal range of HCO3-?

A

23-30

How well did you know this?
1
Not at all
2
3
4
5
Perfectly
5
Q

what is the normal range of Ca2+?

A

8.5-11

How well did you know this?
1
Not at all
2
3
4
5
Perfectly
6
Q

what is the normal range of Mg2+?

A

around 2

How well did you know this?
1
Not at all
2
3
4
5
Perfectly
7
Q

what is normal SCr level?

A

around 1

How well did you know this?
1
Not at all
2
3
4
5
Perfectly
8
Q

what is the normal range of BUN?

A

8-20

How well did you know this?
1
Not at all
2
3
4
5
Perfectly
9
Q

what is the approximate level of glucose that would be considered too high?

A

over 200

How well did you know this?
1
Not at all
2
3
4
5
Perfectly
10
Q

what is the normal range of hemoglobin A1C?

A

below 5.7%

How well did you know this?
1
Not at all
2
3
4
5
Perfectly
11
Q

what A1C level would indicate diabetes?

A

6.5% or more

How well did you know this?
1
Not at all
2
3
4
5
Perfectly
12
Q

A 45-year-old patient comes to the emergency department
because 5-days history of worsening swelling in his legs and
numbness in his hands and feet. He first noticed the
symptoms 5 years ago, and they have continued to gradually
worsen.
His temperature is 37°C (98.7°F), blood pressure is 165/105
mmHg, pulse is 60/min, respirations are 16/min.
He has smoked 20 cigarettes daily for the past 20 years. A
repeat blood pressure reading 5 minutes later shows 170/105
mmHg. The patient appears to be alert and responsive to
questioning. Physical examination shows 2+ pitting edema in
his lower extremities and 1+ pitting edema in the upper
extremities. Neurologic examination shows decreased
sensation to touch in both hands and forearms, and feet and
ankles. Laboratory studies show normal values except for Cr of 4, urea nitrogen 50, glucose 350, and A1C 9%

Based on this information, the patient is likely suffering from? what diagnosis would you give this patient? is their condition reversible? what is the treatment plan?

A

neuropathy due to type 2 diabetes which is irreversible. Treatment is to slow progression and reduce edema

How well did you know this?
1
Not at all
2
3
4
5
Perfectly
13
Q

A 55-year-old patient comes to the emergency department because of a 2-day
history of decreased urinary frequency. He was hospitalized for an episode of acute
pyelonephritis 2 weeks ago and was treated empirically with ceftriaxone and
gentamicin. His symptoms and laboratory abnormalities were completely resolved
by day 5 of treatment and he was discharged from the hospital 10 days ago. He says
that since his discharge he has been drinking lots of water but has not yet urinated.
He has no other known medical history and does not take any medications. His
temperature is 37°C (98.7°F), blood pressure is 152/98 mmHg, pulse is 60/min, and
respirations are 19/min. Physical examination shows 3+ pitting edema in both lower
extremities and bibasilar crackles in the lungs. There is no flank pain or abdominal
tenderness. A basal metabolic panel and complete blood count, urinalysis, and
glomerular filtration rate on his day of hospital discharge were within normal limits.
***pts potassium level is 6

What is acute pyelonephritis? what adverse reactions from gentamicin may be causing the pts inability to urinate? What could explain the pts extremely high BP if he has no known medical history? what is the diagnosis/treatment plan?

A

bacterial kidney infection causing inflammation.
Gentamicin is renal and ototoxic, so it may be damaging the pts kidney function.
kidney function affects BP, so if kidney is damaged it will impact the bodies BP.
Acute nephrotoxicity due to gentamicin. pt needs dialysis STAT to decrease potassium levels. patient needs to start a diuretic to reduce edema, but it won’t work effectively if kidney is damaged.

How well did you know this?
1
Not at all
2
3
4
5
Perfectly
14
Q

T/F patients with AKI are more likely to die prematurely after hospitalization unless their renal function returns to baseline within 7 days

A

False, mortality increases even if renal function returns to baseline

How well did you know this?
1
Not at all
2
3
4
5
Perfectly
15
Q

*slide 11-15 know anatomy

A

.

How well did you know this?
1
Not at all
2
3
4
5
Perfectly
16
Q

A 65-year-old man goes to his primary care physician because of trouble initiating
urination, a problem that has slowly progressed over the last 6 months. A digital
rectal examination reveals a uniformly large, rubbery mass. This patient’s urinary
problems are likely due to which of these?
A. Compression of the bladder
B. Compression of the external urethral sphincter
C. Compression of the membranous urethra
D. Compression of the penile urethra
E. Compression of the prostatic urethra

A

E

How well did you know this?
1
Not at all
2
3
4
5
Perfectly
17
Q

kidneys receive what percentage of cardiac output at rest?

A

25%

How well did you know this?
1
Not at all
2
3
4
5
Perfectly
18
Q

where does most reabsorption occur in the kidney?

A

at the proximal convoluted tubule

How well did you know this?
1
Not at all
2
3
4
5
Perfectly
19
Q

the afferent arteriole ______ blood whereas the efferent arteriole ______ blood

A

brings, takes

How well did you know this?
1
Not at all
2
3
4
5
Perfectly
20
Q

what vitamin does the kidney help produce which helps with calcium absorption?

A

vitamin D

How well did you know this?
1
Not at all
2
3
4
5
Perfectly
21
Q

if the kidney is damaged and produces less vitamin D, what is the main risk factor?

A

increases risk of bone disease

How well did you know this?
1
Not at all
2
3
4
5
Perfectly
22
Q

what hormone does the kidney produce which has the primary role of increasing RBCs?

A

erythropoietin

How well did you know this?
1
Not at all
2
3
4
5
Perfectly
23
Q

what does high glucose in the urine indicate?

A

high glucose plasma concentrations (insulin impairment)

How well did you know this?
1
Not at all
2
3
4
5
Perfectly
24
Q

if a patient is admitted with edema and foamy urine, what is likely occurring?

A

more protein than normal is in the urine which leads to decreased plasma protein causing liquid to leak (edema)

How well did you know this?
1
Not at all
2
3
4
5
Perfectly
25
Q

What is the term used to describe a substance moving from a peritubular capillary into the renal tubule?
A. Reabsorption
B. Excretion
C. Filtration
D. Secretion

A

D

How well did you know this?
1
Not at all
2
3
4
5
Perfectly
26
Q

Which of the following components of a nephron perform the function of filtration?
A. Glomerulus
B. Proximal convoluted tubule
C. Medulla
D. Thin ascending limb
E. Thin descending limb

A

A

How well did you know this?
1
Not at all
2
3
4
5
Perfectly
27
Q

sodium reabsorption occurs in the ____ limb of the loop of henle

A

ascending

How well did you know this?
1
Not at all
2
3
4
5
Perfectly
28
Q

water reabsorption occurs in the ____ limb of the loop of henle

A

descending

How well did you know this?
1
Not at all
2
3
4
5
Perfectly
29
Q

A patient presents to his physician with a 3-week history of frothy urine as well as
edema. He also has a history of chronic hepatitis B. A urine dipstick is positive for
protein, and a 24-hour urine shows 4 g of protein per day. A renal biopsy demonstrates
subepithelial deposits with a spike and dome appearance. The positive urine dipstick for
protein is most likely due to damage to what part of the glomerulus?
A. Bowman capsule
B. Capillary endothelial cells
C. Glomerular basement membrane
D. Juxtaglomerular cells
E. Macula densa

A

C

How well did you know this?
1
Not at all
2
3
4
5
Perfectly
30
Q

the first layer of the glomerulus is also called ___________ and prevents what from passing through?

A

endothelium, blood…albumin

31
Q

the second layer of the glomerulus is called ___________ and prevents what from passing through?

A

Glomerular basement membrane
protein

32
Q

the third layer of the glomerulus is called ___________ and prevents what from passing through?

A

Epithelium
almost everything but water

33
Q

An otherwise healthy 27-year-old patient presents to the emergency department with
dark urine and left flank pain. He has had a fever, sore throat, and malaise for the last 2
days. The vital signs include: temperature 38.1°C (100.5°F), blood pressure 120/82 mm
Hg, and pulse 95/min. His family history is noncontributory. Physical examination
reveals enlarged tonsils with tender anterior cervical lymphadenopathy. Urinalysis
shows pink urine with 20–25 red blood cells/high power field (RBCs/HPF) and 2+
protein. This patient’s condition is most likely due to which of the following?
A. Inherited abnormalities in type IV collagen
B. Autoantibodies against alpha-3 chain of type IV collagen
C. C3 nephritic factor
D. Immune complex deposition
E. Diffuse mesangial IgA deposition

A

D

34
Q

The final components of urine do NOT include? Select-ALL-that apply:*
A. Glucose
B. Urea
C. Creatinine
D. Lipids
E. Proteins

A

A,D,E

35
Q

what is the autoregulatory range of mean arterial blood pressure for maintaining normal renal blood flow?

A

80-180

36
Q

what effect does angiotensin 2 have one renal circulation?
what effect does angiotensin 2 have on RBF and GFR?

A

causes vasoconstriction of efferent > afferent arteriole
decreases RBF and increases GFR

37
Q

what effect does norepi and epi have on renal circulation?
what effect does norepi and epi have on RBF and FGR?

A

vasoconstriction of afferent and efferent arterioles
decreases both

38
Q

what effect does natriuretic peptides (ANP, BNP) have on renal circulation?
what effect does natriuretic peptides (ANP, BNP) have on RBF and GFR?

A

vasodilates afferent arteriole and constricts efferent arteriole
no change to RBF, increases GFR

39
Q

what effect do prostaglandins have on renal circulation?
what effect do prostaglandins have on RBF and GFR?

A

vasodilation of afferent > efferent arteriole
increases both

40
Q

what is the myogenic response for autoregulating GFR and RBF?

A

if RBF increases, then pressure against the walls of the afferent arteriole increases. the response causes stretch receptors in smooth muscle to initiate vasoconstriction. this response decreases RBF which will decrease the pressure and decrease GFR

41
Q

when tubule flow increases, what is the autoregulation process that causes it to decrease?

A

with increased tubule flow, more NaCl will enter. Adenosine senses high sodium to cause afferent constriction and efferent dilation to decrease pressure and GFR which decreases tubule flow. the increase of NaCl also reduces renin release which blocks angiotensin 2 (the hormone which prevents efferent constriction)

42
Q

when tubule flow decreases, what is the autoregulation process that causes it to increase?

A

with decreased tubule flow, less NaCl will enter. this blocks adenosine and increases renin release to produce more angiotensin 2 to cause efferent constriction. this then increases pressure and increases GFR to increase tubule flow

43
Q

overall, the net result of autoregulation keeps GFR and tubule flow constant at all times, what are the simultaneous processes that keep GFR constant?

A

when afferent pressure decreases due to constriction, GFR falls. however, efferent pressure also decreases due to constriction, and GFR increases. on the opposite, when afferent pressure increases due to dilation, GFR increases. however, efferent pressure also increases due to dilation, and GFR falls

44
Q

Which of the following best describes the relationship between renal flow rate and mean
arterial blood pressure within the autoregulatory zone?
A. Increased flow, when mean arterial blood pressure increases
B. Increased flow, when mean arterial blood pressure decreases
C. Decreased flow, when mean arterial blood pressure decreases
D. Constant flow, regardless of changes in mean arterial blood pressure
E. Decreased flow, when mean arterial blood pressure increases

A

D

45
Q

Which of the following will be released after a high concentration of NaCl is sensed by the
macula densa?
A. Renin
B. Adenosine
C. Angiotensin II
D. Angiotensin I
E. Aldosterone

A

B

46
Q

A research study investigates the effects of various medications on the renal
vasculature, focusing on the afferent and efferent arteriole. Which of the following
most accurately describes the effect of lisinopril [ACE-I] on the renal vasculature?
A. Constricts afferent and efferent arteriole
B. Constricts afferent arteriole and dilates efferent arteriole
C. Constricts afferent arteriole and no change in efferent arteriole
D. Dilates afferent arteriole and constricts efferent arteriole
E. Dilates both the afferent and efferent arteriole, with preferential dilation of
the efferent arteriole
F. No change in afferent arteriole, constricts efferent arteriole

**explain the answer

A

E
An ACE inhibitor such as lisinopril prevents the conversion of angiotensin 1 to angiotensin 2. since angiotensin 2 causes constriction of both the efferent and afferent arterioles, they will dilate if it is not being produced.

47
Q

Which of the following hormones is a vasoconstrictor?
A. Nitric oxide
B. Natriuretic hormone
C. Bradykinin
D. Angiotensin II

A

D

48
Q

What signal causes the heart to secrete atrial natriuretic hormone?
A. Increased blood pressure
B. Decreased blood pressure
C. Increased Na+ levels
D. Decreased Na+ levels

A

D

49
Q

blood pressure is primarily maintained by the balanced function of what 3 systems?

A

The Renin-Angiotensin-Aldosterone (RAA) cycle
Production of Atrial Natriuretic Peptides
Anti-Diuretic Hormone/Vasopressin (ADH/VP)

50
Q

what does the Renin-Angiotensin-Aldosterone (RAA) cycle regulate?

A

the amount of sodium lost and eliminated in urine

51
Q

what does Anti-Diuretic Hormone/Vasopressin (ADH/VP) do?

A

controls water balance by regulating the loss and elimination of water into urine

52
Q

describe the renin-angiotensin system which regulates the production of aldosterone

A

the afferent vessel is stimulated if there is decreased perfusion, low serum sodium, reduced total blood volume, or reduced blood pressure. this triggers the production of renin which is converted to angiotensin 2 to stimulate the adrenal cortex and release aldosterone

53
Q

what is the function of aldosterone?

A

control the balance of water and salts in the kidney by keeping sodium in and releasing potassium from the body

54
Q

what three things trigger an increased production of aldosterone?

A

decreased blood pressure
decreased renal flow
hyperkalemia

55
Q

what is an important adverse reaction of an aldosterone antagonist that should be monitored?

A

hyperkalemia

56
Q

what drug is an aldosterone antagonist? how does it work?

A

spironolactone
it is a potassium sparing diuretic that can decrease blood pressure, treat edema, and treat heart failure

57
Q

Which hormone plays a role in water reabsorption in the collecting duct/tubule?
A. Aldosterone
B. Thyroxine
C. Anti-diuretic hormone
D. Corticotropin-releasing hormone

A

C

58
Q

what drugs are SGLT-2 inhibitors and what is their mechanism of action?

A

gliflozins or flozins
they block reabsorption of sodium and glucose to decrease blood glucose, reduce blood pressure, and prevent CVD

59
Q

where are SGLT-2s located?

A

at the renal proximal tubule

60
Q

what is the mechanism of action of ADH?

A

it binds to the V2 receptor and increases the number of aquaporins which increases water reabsorption

61
Q

what do aquaretics block and why are they not typically used?

A

they block V2 receptors. they are too potent and cause too much uncontrolled water loss

62
Q

what are the names of 2 two V2 selective antagonist drugs?

A

conivaptan and tolvaptan

63
Q

A patient with chronic kidney disease [CKD] was admitted for Hgb of 7 gm/dl and HCT of
21%. What is the cause of anemia seen in patients with CKD?
A. Decreased erythropoietin production
B. Excessive sequestration of red blood cells in the spleen
C. Vitamin deficiency
D. Red cell membranopathy
E. Deficient globin synthesis

A

A

64
Q

what is approximately normal hematocrit levels?

A

40%

65
Q

how does the sympathetic nervous system respond to decline in systolic function?

A

epi and norepi is released via adrenergic receptors to cause:
vasoconstriction, increased heart rate, and increased contractility

66
Q

how does the renin angiotensin aldosterone system respond to decline in systolic function?

A

angiotensin 2 is released to cause:
vasoconstriction, increased BP, increased aldosterone, and increased fibrosis (thickening/scarring of heart tissue)

67
Q

how does the natriuretic peptide system respond to decline in systolic function?

A

NPs released to cause:
vasodilation, decreased BP, decreased vasopressin, decreased aldosterone, and decreased fibrosis

68
Q

what drug classes do we use to treat heart failure?

A

Beta blockers (reduce heart rate and contractility), ACE-Is (block vasoconstriction), and enhancing the natriuretic peptide system

69
Q

what is the mechanism of action of entresto? (sacubitril/valsartan)

A

blocks conversion of angio 1 to angio 2 and blocks neprilysin from breaking down natriuretic peptides

70
Q

A marathon runner drinks a large quantity of electrolyte solution (sports drink) with
expectations to improve performance during races. Elevation of what electrolyte in his
urine is responsible for changes in tubuloglomerular feedback?
A. Chloride
B. Glucose
C. Magnesium
D. Potassium
E. Urea

A

The correct answer is (A), chloride. In tubuloglomerular feedback, if there is increased
sodium and chloride sensed by the macula densa cells at the distal convoluted tubule,
this will cause a reflexive constriction of the afferent arteriole, decreasing renal blood
flow and thus the glomerular filtration rate. The other answers—glucose (B),
magnesium (C), potassium (D) and urea (E)—are not involved in tubuloglomerular
feedback.

71
Q

A 48-year-old man comes in to your clinic with lightheadedness. He says he just finished
a week-long hike and ran out of water yesterday. His blood pressure is 80/60 mmHg.
Recalling the renal physiology you learned, you remember the kidney has a cell that
responds to low blood pressure by secreting renin. Which of the following best
describes the origin of the cell responsible for secreting renin?
A. Endothelium
B. Epithelium
C. Modified smooth muscle
D. Neuron
E. Skeletal muscle

A

The correct answer is (C), modified smooth muscle. The juxtaglomerular (JG) cells are of
smooth muscle origin and they reside in the juxtaglomerular apparatus, a section of the
nephrons where the ascending limb of the loop of Henle meets with the afferent and
efferent arterioles. It is these JG cells that respond to low blood pressure by secreting
renin. Endothelial cells (A) can secrete nitric oxide, a vasodilator, or they can secrete
endothelin, a vasoconstrictor. JG cells are not epithelial cells (B), neuronal cells (D), or
skeletal muscle cells (E).

72
Q

A 70-year-old woman with chronic osteoarthritis is being seen in the hospital. She has
had trouble urinating, she is dehydrated, and her glomerular filtration rate (GFR) is very
low. She takes an ACE (angiotensin converting enzyme) inhibitor for hypertension. Given
her history, what is most likely cause of her low GFR?
A. Constriction of the afferent arteriole
B. Constriction of the afferent and efferent arterioles
C. Constriction of the efferent arteriole
D. Dilation of the afferent arteriole
E. Dilation of the efferent arteriole

A

The correct answer is (E), dilation of the efferent arteriole. ACE inhibitors preferentially
dilate the efferent arteriole. This is good when we are trying to avoid high glomerular
pressures in chronic kidney disease but can lead to a reduced GFR when patients get
dehydrated and need the high efferent tone to maintain GFR. ACE inhibitors do not act
significantly on the afferent arterioles (A, B, D). They dilate, not constrict, the efferent
arteriole (C).

73
Q

While working in a renal physiology lab, you decide to evaluate the effects of
physiologic doses of angiotensin 2 by injecting it into subject lab rats and measuring
renal blood flow (RBF). Which of the following is true regarding the effects of
angiotensin 2 on renal vasculature?
A. At low doses, it is a vasodilator of afferent and efferent arterioles. At high doses, it is
a vasoconstrictor.
B. No change
C. Vasoconstriction, preferential vasoconstriction of efferent arteriole with increased
filtration fraction
D. Vasodilates afferent arteriole, constricts efferent arteriole, increasing filtration
fraction
E. Vasodilation, preferential vasodilation of afferent arteriole with no increase in
filtration fraction

A

The correct answer is (C), preferential vasoconstriction of the efferent arteriole with
increased filtration fraction. Angiotensin 2 will preferentially constrict the efferent
arteriole, which will cause a decrease in RBF but an increase in GFR. This combination
will lead to an increased filtration fraction. Option (A) describes dopamine. Angiotensin
2 will change RBF and GFR, making choice (B), no change, incorrect. Option (D)
describes ANP. Option (E) describes prostaglandins.